Đến nội dung

Hình ảnh

Inequalities From 2016 Mathematical Olympiads

* * * * * 3 Bình chọn

  • Please log in to reply
Chủ đề này có 139 trả lời

#21
dark templar

dark templar

    Kael-Invoker

  • Hiệp sỹ
  • 3788 Bài viết

 

Bài 12 (China Junior High School Mathematics League). Với $x,\,y,\,z$ là ba số thực dương thỏa mãn $xy+yz+zx\neq 1$ và $\frac{(x^2-1)(y^2-1)}{xy} +\frac{(y^2-1)(z^2-1)}{yz} +\frac{(z^2-1)(x^2-1)}{zx} =4.$ Prove that $$9(x+y)(y+z)(z+x)\geqslant 8xyz(xy+yz+zx).$$

Biến đổi điều kiện đề bài cho ,ta sẽ có:

$\sum x\left ( y^{2}-1 \right )\left ( z^{2}-1 \right )=4xyz\Leftrightarrow xyz\sum xy+\sum x-\sum xy(x+y)=4xyz$

$\Leftrightarrow xyz\sum xy+\left ( \sum x-xyz \right )=\left ( \sum x \right )\left ( \sum xy \right )\Leftrightarrow \left ( \sum xy -1 \right )\left ( \sum x -xyz\right )=0$

$\Leftrightarrow x+y+z=xyz$

 

Đổi biến $\left ( a,b,c \right )\rightarrow \left ( \frac{1}{x},\frac{1}{y},\frac{1}{z} \right )$ thì ta có $a,b,c>0$ và $ab+bc+ca=1$.

 

BĐT cần chứng minh trở thành:

$$9(a+b)(b+c)(c+a)\geqslant 8(a+b+c)$$

 

Viết dưới dạng thuần nhất :

$$\left ( a+b \right )(b+c)(c+a)\geqslant \frac{8}{9}(a+b+c)(ab+bc+ca)$$

 

BĐT này chỉ là hệ quả của $(a+b)(b+c)(c+a)=(a+b+c)(ab+bc+ca)-abc$ và $abc\leqslant \frac{(a+b+c)(ab+bc+ca)}{9}$.Ta có đpcm.


"Do you still... believe in me ?" Sarah Kerrigan asked Jim Raynor - Starcraft II:Heart Of The Swarm.

#22
tungteng532000

tungteng532000

    Trung sĩ

  • Thành viên
  • 174 Bài viết

 

 

Bài 12 (China Junior High School Mathematics League). Với $x,\,y,\,z$ là ba số thực dương thỏa mãn $xy+yz+zx\neq 1$ và $\frac{(x^2-1)(y^2-1)}{xy} +\frac{(y^2-1)(z^2-1)}{yz} +\frac{(z^2-1)(x^2-1)}{zx} =4.$ Prove that $$9(x+y)(y+z)(z+x)\geqslant 8xyz(xy+yz+zx).$$

Đặt a+b+c=p; ab+bc+ca=q; abc=r khi đó giả thiết tương đương với:
$qr-pq+p-r=0\Leftrightarrow (r-p)(q-1)=0\Leftrightarrow p=r$ vì $q\neq 1$
Bđt cần chứng minh trở thành:
$9(x+y)(y+z)(z+x)\geq 8(x+y+z)(xy+yz+zx)\Leftrightarrow \sum x(y-z)^2\geq 0$ (đúng)
Đẳng thức xảy ra khi x=y=z= $\sqrt{3}$


                                              Lời giải hay thì like nhé :))
FB: 
https://www.facebook...oylanh.lung.564


#23
Nguyenhuyen_AG

Nguyenhuyen_AG

    Trung úy

  • Thành viên nổi bật 2016
  • 945 Bài viết

p.s nhờ anh huyện sửa bài em thành latex nha em dùng dtdd nên ko viết latex dc

 

Anh không phải DHV nên không có quyền can thiệp vào bài viết của các thành viên khác nhưng em cứ viết bằng di động khi nào online bằng điện thoại thì em vào bài của mình để sửa lại cũng được chứ nhỉ. :)


Bài viết đã được chỉnh sửa nội dung bởi Nguyenhuyen_AG: 05-04-2016 - 23:12

Nguyen Van Huyen
Ho Chi Minh City University Of Transport

#24
NTA1907

NTA1907

    Thượng úy

  • Thành viên
  • 1014 Bài viết

Bài 3 (Korea Winter Program Practice Test). Cho ba số thực không âm $x,\,y,\,z$ thỏa mãn

\[(x+y-1)^2+(y+z-1)^2+(z+x-1)^2=27.\]
Tìm giá trị lớn nhất của biểu thức $x^4+y^4+z^4.$

Bài này đề gốc là tìm cả min cả max chứ anh?

302f2d34349b033bf9410dd512ce36d3d439bdfe.jpg


Vũ trụ không có biên trong không gian, không có bắt đầu và kết thúc trong thời gian và chẳng có việc gì cho đấng sáng thế phải làm ở đây cả.

 


#25
Nguyenhuyen_AG

Nguyenhuyen_AG

    Trung úy

  • Thành viên nổi bật 2016
  • 945 Bài viết

13)(koreanMO2016 ngày2 )cho $x,y,z$ là số thực thỏa $x^2+y^2+z^2=1$, tìm max $$P=(x^2-yz)(y^2-zx)(z^2-xy)$$

 

Cho $x=\frac{1}{\sqrt{2}},y=-\frac{1}{\sqrt{2}},z=0$ thì $P = \frac{1}{8}$ do đó nếu ta chứng minh được

\[8(x^2-yz)(y^2-zx)(z^2-xy) \leqslant (x^2+y^2+z^2)^3,\]

thì $\frac{1}{8}$ chính là giá trị lớn nhất cần tìm.

 

Ta thấy rằng rằng trong ba số $x^2-yz,y^2-zx,z^2-xy$ sẽ có ít nhất hai số cùng dấu. Giả sử $(x^2-yz)(y^2-zx) \geqslant 0,$ khi đó

  • Nếu $z^2-xy \leqslant 0$ thì \[8(x^2-yz)(y^2-zx)(z^2-xy) \leqslant 0 \leqslant  (x^2+y^2+z^2)^3.\]
  • Nếu $z^2-xy \geqslant 0$ theo bất đẳng thức AM-GM, ta được \[4(x^2-yz)(y^2-zx) \leqslant (x^2-yz+y^2-zx)^2.\] Cho nên ta chỉ cần chỉ ra \[(x^2+y^2+z^2)^3 \geqslant 2(x^2-yz+y^2-zx)^2(z^2-xy),\] thu gọn thành \[\left [2(x^2+y^2)(z^2-xy)+(x^2+y^2+z^2)(x+y-z)^2 \right ](x+y+z)^2 \geqslant 0.\] Hiển nhiên đúng.

Từ đó dẫn đến kết luận của bài toán.

 

Bài này đề gốc là tìm cả min cả max chứ anh?

 

Cám ơn em, anh đã cập nhật.


Bài viết đã được chỉnh sửa nội dung bởi Nguyenhuyen_AG: 05-04-2016 - 23:26

Nguyen Van Huyen
Ho Chi Minh City University Of Transport

#26
Nguyenhuyen_AG

Nguyenhuyen_AG

    Trung úy

  • Thành viên nổi bật 2016
  • 945 Bài viết

Cập nhật một số bài mới.

 

Bài 14 (Turkey TST). Với $a,\,b,\,c$ là ba số thực không âm thỏa mãn điều kiện $a^2+b^2+c^2 \leqslant 3.$ Chứng minh rằng
$$(a+b+c)(a+b+c-abc)\ge2(a^2b+b^2c+c^2a).$$

Bài 15 (Australien MO). Cho $a,b$ là hai số thực thỏa mãn $a^{2}+b^{2}=1.$ Chứng minh rằng
$$\left | a+\frac{a}{b}+b+\frac{b}{a} \right |\geq 2-\sqrt{2}.$$

Bài 16 (Israel Winter Camp). Với $a,\,b,\,c$ là ba số thực bất kỳ. Chứng minh rằng
\[4(a^6+b^6+c^6)+5abc(a^3+b^3+c^3)\geq(ab+ac+bc)^3.\]


Bài viết đã được chỉnh sửa nội dung bởi Nguyenhuyen_AG: 06-04-2016 - 01:36

Nguyen Van Huyen
Ho Chi Minh City University Of Transport

#27
Gachdptrai12

Gachdptrai12

    Thượng sĩ

  • Điều hành viên THCS
  • 280 Bài viết

câu 14 đã có ở đây https://www.artofpro...imo_tst_2016_p3 với lời giải của quykhtn



#28
IMOer

IMOer

    Hạ sĩ

  • Thành viên
  • 50 Bài viết

Bài 4 (Romanian Masters In Mathematics). Cho hai số thực dương $x,\,y$ thỏa mãn $x+y^{2016}\geqslant 1.$ Chứng minh rằng $$x^{2016}+y> 1-\frac{1}{100}.$$

 

Nếu $x\ge 1-\frac{1}{100\cdot 2016}$, áp dụng bất đẳng thức Bernoulli ta có:

${{x}^{2016}}\ge {{\left( 1-\frac{1}{100\cdot 2016} \right)}^{2016}}>1-2016\cdot \frac{1}{100\cdot 2016}=1-\frac{1}{100}$

Nên suy ra: ${{x}^{2016}}+y>\frac{1}{100}$

Nếu $x<1-\frac{1}{100\cdot 2016}$ thì ta có: $y\ge {{\left( 1-x \right)}^{\frac{1}{2016}}}>{{\left( 100\cdot 2016 \right)}^{-\frac{1}{2016}}}$.

Ta chỉ cần chứng minh: ${{\left( 100\cdot 2016 \right)}^{-\ \frac{1}{2016}}}>1-\frac{1}{100}\Leftrightarrow {{\left( 1+\frac{1}{99} \right)}^{2016}}>100\cdot 2016$

Thật vậy, áp dụng bất đẳng thức Bernoulli, ta có:

${{\left( 1+\frac{1}{99} \right)}^{2016}}>{{\left( 1+\frac{1}{99} \right)}^{99\cdot 20}}>{{\left( 1+99\cdot \frac{1}{99} \right)}^{20}}={{2}^{20}}>100\cdot 2016$

Vậy bất đẳng thức đã được chứng minh.



#29
Nguyenhuyen_AG

Nguyenhuyen_AG

    Trung úy

  • Thành viên nổi bật 2016
  • 945 Bài viết
Bài 16 (Israel Winter Camp). Với $a,\,b,\,c$ là ba số thực bất kỳ. Chứng minh rằng

\[4(a^6+b^6+c^6)+5abc(a^3+b^3+c^3)\geq(ab+ac+bc)^3.\]

 

Đặt $p=a+b+c,\,q=ab+bc+ca$ và $r=abc$ khi đó bất đẳng thức cần chứng minh trở thành
\[27r^2+(29p^3-63pq)r+4p^6-24p^4q+36p^2q^2-9q^3 \geqslant 0. \quad (1)\]
Giả sử $p=3$ và đặt $q=3-3t^2\;(0 \leqslant t < 1)$ thì $(1)$ tương đương với
\[r^2+(21t^2+8)r+9t^6+81t^4+27t^2-9 \geqslant 0.\]
Đặt \[f(r) = r^2+(21t^2+8)r+9t^6+81t^4+27t^2-9,\] ta sẽ chứng minh $f(r) \geqslant 0.$

Thật vậy nếu $0 \leqslant t \leqslant \frac{1}{2}$ thì do $r \geqslant (1-2t)(1+t)^2 \geqslant 0$ cho nên
\[f(r) \geqslant f[(1-2t)(1+t)^2] = t^2(13t^4-30t^3+27t^2-20t+18).\]
Lại đặt
\[f(t) = 13t^4-30t^3+27t^2-20t+18.\]
Ta có
\[f^{'}(t) = 2(26t^3-45t^2+27t-10),\]
nên
\[f^{'}(t) = 0 \Leftrightarrow  t_0 = \frac{15}{26}+\frac{1}{26}\left ( \sqrt[3]{1490+13\sqrt{13141}}-\frac{9}{\sqrt[3]{1490+13\sqrt{13141}}} \right ).\]
Suy ra
\[f(t) \geqslant \left \{ f(0),\,f(t_0),\,f\left(\frac{1}{2}\right) \right \} = f(t_0) > 0.\]
Còn nếu $\frac{1}{2} \leqslant t < 1,$ thì
\[f(r) > 9t^6+81t^4+27t^2-9 > 9\left ( \frac{1}{2} \right )^6+81\left ( \frac{1}{2} \right )^4+27\left ( \frac{1}{2} \right )^2-9 = \frac{189}{64} > 0.\]
Vậy $f(r) > 0.$ Bài toán được chứng minh.


Nguyen Van Huyen
Ho Chi Minh City University Of Transport

#30
Nguyenhuyen_AG

Nguyenhuyen_AG

    Trung úy

  • Thành viên nổi bật 2016
  • 945 Bài viết

Bài 17 (Azerbaijan Junior Mathematical Olympiad). Với $x,\,y,\,z$ là ba số thực khác $0.$ Chứng minh rằng $$\sqrt {x^2+\frac {1}{y^2}}+ \sqrt {y^2+\frac {1}{z^2}}+ \sqrt {z^2+\frac {1}{x^2}}\geq 3\sqrt {2}. $$


Nguyen Van Huyen
Ho Chi Minh City University Of Transport

#31
Gachdptrai12

Gachdptrai12

    Thượng sĩ

  • Điều hành viên THCS
  • 280 Bài viết

Đặt $p=a+b+c,\,q=ab+bc+ca$ và $r=abc$ khi đó bất đẳng thức cần chứng minh trở thành
\[27r^2+(29p^3-63pq)r+4p^6-24p^4q+36p^2q^2-9q^3 \geqslant 0. \quad (1)\]
Giả sử $p=3$ và đặt $q=3-3t^2\;(0 \leqslant t < 1)$ thì $(1)$ tương đương với
\[r^2+(21t^2+8)r+9t^6+81t^4+27t^2-9 \geqslant 0.\]
Đặt \[f(r) = r^2+(21t^2+8)r+9t^6+81t^4+27t^2-9,\] ta sẽ chứng minh $f(r) \geqslant 0.$

Thật vậy nếu $0 \leqslant t \leqslant \frac{1}{2}$ thì do $r \geqslant (1-2t)(1+t)^2 \geqslant 0$ cho nên
\[f(r) \geqslant f[(1-2t)(1+t)^2] = t^2(13t^4-30t^3+27t^2-20t+18).\]
Lại đặt
\[f(t) = 13t^4-30t^3+27t^2-20t+18.\]
Ta có
\[f^{'}(t) = 2(26t^3-45t^2+27t-10),\]
nên
\[f^{'}(t) = 0 \Leftrightarrow t_0 = \frac{15}{26}+\frac{1}{26}\left ( \sqrt[3]{1490+13\sqrt{13141}}-\frac{9}{\sqrt[3]{1490+13\sqrt{13141}}} \right ).\]
Suy ra
\[f(t) \geqslant \left \{ f(0),\,f(t_0),\,f\left(\frac{1}{2}\right) \right \} = f(t_0) > 0.\]
Còn nếu $\frac{1}{2} \leqslant t < 1,$ thì
\[f(r) > 9t^6+81t^4+27t^2-9 > 9\left ( \frac{1}{2} \right )^6+81\left ( \frac{1}{2} \right )^4+27\left ( \frac{1}{2} \right )^2-9 = \frac{189}{64} > 0.\]
Vậy $f(r) > 0.$ Bài toán được chứng minh.

anh huyện bài ni nếu áp dụng pp ABC thì f(r) có bậc 2 nên nó đạt đuợc cực trị khi có 1 biến bằng 0 hoặc 2 biến bằng nhau
nên ta có thể cho a=b.Mà bđt đồng bậc chuẩn hóa a=1 thì f© >=0 ta có đpcm
p.s ko biết đúng ko. à mà khúc q=3-3t^2 là đặt a^2+b^2+c^2=3+6t^2 hả anh :))

Bài viết đã được chỉnh sửa nội dung bởi Gachdptrai12: 08-04-2016 - 21:01


#32
Element hero Neos

Element hero Neos

    Trung úy

  • Thành viên
  • 943 Bài viết

Bài 17 (Azerbaijan Junior Mathematical Olympiad). Với $x,\,y,\,z$ là ba số thực khác $0.$ Chứng minh rằng $$\sqrt {x^2+\frac {1}{y^2}}+ \sqrt {y^2+\frac {1}{z^2}}+ \sqrt {z^2+\frac {1}{x^2}}\geq 3\sqrt {2}. $$

Áp dụng các bất đẳng thức Minkowsky, C.B.S và AM-GM có: 

$\sqrt{x^2+\frac{1}{y^2}}+\sqrt{y^2+\frac{1}{z^2}}+\sqrt{z^2+\frac{1}{x^2}}\geq\sqrt{(x+y+z)^2+(\frac{1}{x}+\frac{1}{y}+\frac{1}{z})^2}\geq\sqrt{(x+y+z)^2+(\frac{9}{x+y+z})^2}\geq 3\sqrt{2}(Q.E.D)$


Bài viết đã được chỉnh sửa nội dung bởi Element hero Neos: 08-04-2016 - 21:10


#33
Nguyenhuyen_AG

Nguyenhuyen_AG

    Trung úy

  • Thành viên nổi bật 2016
  • 945 Bài viết
Bài 11 (Silk Road Mathematical Olympiad). Với $a,\,b,\,c$ là ba số thực thỏa mãn $| (a-b) (b-c) (c-a) | = 1.$ Tìm giá trị nhỏ nhất của $| a | + | b | + | c |.$

 

Giả sử $a>b>c$ khi đó theo bất đẳng thức AM-GM, ta có

\[1= (a-c)\cdot(a-b)(b-c) \leqslant (a-c) \cdot \frac{1}{4}\left [ (a-b)+(b-c) \right ]^2=\frac{1}{4}(a-c)^3.\]

Do đó $a-c \geqslant \sqrt[3]{4}.$ Mặt khác

\[| a | + | b | + | c | \geqslant |a|+|-c| \geqslant |a-c| \geqslant \sqrt[3]{4}.\]

Đẳng thức xảy ra khi $a=\frac{\sqrt[3]{4}}{2},b=0,c=-\frac{\sqrt[3]{4}}{2}.$ Vậy giá trị nhỏ nhất cần tìm là $\sqrt[3]{4}.$


Bài viết đã được chỉnh sửa nội dung bởi Nguyenhuyen_AG: 08-04-2016 - 21:48

Nguyen Van Huyen
Ho Chi Minh City University Of Transport

#34
Nguyenhuyen_AG

Nguyenhuyen_AG

    Trung úy

  • Thành viên nổi bật 2016
  • 945 Bài viết

$$\sqrt{(x+y+z)^2+(\frac{1}{x}+\frac{1}{y}+\frac{1}{z})^2}\geq\sqrt{(x+y+z)^2+(\frac{9}{x+y+z})^2}$$

Bất đẳng thức này sai nếu $x=-1,\,y=\frac{1}{3},\,z=\frac{1}{2}.$


Nguyen Van Huyen
Ho Chi Minh City University Of Transport

#35
Gachdptrai12

Gachdptrai12

    Thượng sĩ

  • Điều hành viên THCS
  • 280 Bài viết

Bài 17 (Azerbaijan Junior Mathematical Olympiad). Với $x,\,y,\,z$ là ba số thực khác $0.$ Chứng minh rằng $$\sqrt {x^2+\frac {1}{y^2}}+ \sqrt {y^2+\frac {1}{z^2}}+ \sqrt {z^2+\frac {1}{x^2}}\geq 3\sqrt {2}. $$

áp dụng bđt AM-GM ta có $VT\geq \sum \sqrt{\frac{2x}{y}}$

tiếp tục áp dụng AM-GM 1 lần nữa ta có$\sum \sqrt{\frac{2x}{y}}\geq 3\sqrt[3]{\sqrt{\frac{8xyz}{yxz}}}$=$3\sqrt{2}$ 

p/s èo sai rồi 3 số thực tưởng mô


Bài viết đã được chỉnh sửa nội dung bởi Gachdptrai12: 08-04-2016 - 23:22


#36
nangcuong8e

nangcuong8e

    Trung sĩ

  • Thành viên
  • 134 Bài viết

áp dụng bđt AM-GM ta có $VT\geq \sum \sqrt{\frac{2x}{y}}$

tiếp tục áp dụng AM-GM 1 lần nữa ta có$\sum \sqrt{\frac{2x}{y}}\geq 3\sqrt[3]{\sqrt{\frac{8xyz}{yxz}}}$=$3\sqrt{2}$

Dòng màu đỏ cần có đk $a,b,c >0$ để nó có nghĩa.

 

Bài 17 (Azerbaijan Junior Mathematical Olympiad). Với $x,\,y,\,z$ là ba số thực khác $0.$ Chứng minh rằng $$\sqrt {x^2+\frac {1}{y^2}}+ \sqrt {y^2+\frac {1}{z^2}}+ \sqrt {z^2+\frac {1}{x^2}}\geq 3\sqrt {2}. $$

Ta có  $a^3 +b^3+c^3 -3abc =(a+b+c)[\frac{(a-b)^2+(b-c)^2+(c-a)^2}{2}] \geq 0 \Leftrightarrow a+b+c \geq 0$
 Mà $\sum \sqrt[6]{x^2 +\frac{1}{y^2}} > 0$ nên $\sum \sqrt{x^2+\frac{1}{y^2}} \geq 3\sqrt[6]{(x^2+\frac{1}{y^2})(y^2+\frac{1}{z^2})(z^2+\frac{1}{x^2})}$.

Áp dụng Holder: $(x^2+\frac{1}{y^2})(y^2+\frac{1}{z^2})(z^2+\frac{1}{x^2}) \geq (\sqrt[3]{(xyz)^2} +\sqrt[3]{\frac{1}{(xyz)^2}})^3$.

$\Leftrightarrow 3\sqrt[6]{(x^2+\frac{1}{y^2})(y^2+\frac{1}{z^2})(z^2+\frac{1}{x^2})} \geq 3\sqrt{\sqrt[3]{(xyz)^2} +\sqrt[3]{\frac{1}{(xyz)^2}}}$

 Cần chứng minh $3\sqrt{\sqrt[3]{(xyz)^2} +\sqrt[3]{\frac{1}{(xyz)^2}}} \geq 3\sqrt{2}$

hay $\sqrt[3]{(xyz)^2} +\sqrt[3]{\frac{1}{(xyz)^2}} \geq 2$ (điều này đúng theo AM - GM)
 Dấu $"="$ xảy ra khi $x=y=z=1$ hoặc $x=y=z=-1$


Bài viết đã được chỉnh sửa nội dung bởi nangcuong8e: 09-04-2016 - 11:21


#37
Gachdptrai12

Gachdptrai12

    Thượng sĩ

  • Điều hành viên THCS
  • 280 Bài viết

Dòng màu đỏ cần có đk $a,b,c >0$ để nó có nghĩa.

Ta có $a^3 +b^3+c^3 -3abc =(a+b+c)(\frac{(a-b)^2+(b-c)^2+(c-a)^2}{2} \geq 0 \Leftrightarrow a+b+c \geq 0$
Mà $\sum \sqrt[6]{x^2 +\frac{1}{y^2}} > 0$ nên $\sum \sqrt{x^2+\frac{1}{y^2}} \geq 3\sqrt[6]{(x^2+\frac{1}{y^2})(y^2+\frac{1}{z^2})(z^2+\frac{1}{x^2})}$.
Áp dụng Holder: $(x^2+\frac{1}{y^2})(y^2+\frac{1}{z^2})(z^2+\frac{1}{x^2}) \geq (\sqrt[3]{(xyz)^2} +\sqrt[3]{\frac{1}{(xyz)^2}})^3$.
$\Leftrightarrow 3\sqrt[6]{(x^2+\frac{1}{y^2})(y^2+\frac{1}{z^2})(z^2+\frac{1}{x^2})} \geq 3\sqrt{\sqrt[3]{(xyz)^2} +\sqrt[3]{\frac{1}{(xyz)^2}}}$
Cần chứng minh $3\sqrt{\sqrt[3]{(xyz)^2} +\sqrt[3]{\frac{1}{(xyz)^2}}} \geq 3\sqrt{2}$
hay $\sqrt[3]{(xyz)^2} +\sqrt[3]{\frac{1}{(xyz)^2}} \geq 2$ (điều này đúng theo AM - GM)
Dấu $"="$ xảy ra khi $x=y=z=1$ hoặc $x=y=z=-1$

dòng 1 bđt sai nếu a,b chạy tơí âm vô cùng còn c=0

#38
minhrongcon2000

minhrongcon2000

    Thượng sĩ

  • Thành viên
  • 213 Bài viết

Bài 10 (CHKMO). Với $n$ số thực $a_1,\,a_2,\,\ldots,\,a_n$ thuộc $(1,-1)$ và số nguyên $1 \leqslant i \leqslant n$ thỏa mãn điều kiện

  • $a_1+a_2+\cdots+a_n=0.$
  • $a_1^2+a_2^2+\cdots+a_n^2=40.$

Tìm giá trị nhỏ nhất của $n.$

Bài giải

Do $a_{i} \in (-1;1)$ và $\sum_{i=1}^{n} a_{i}^{2}=40$ nên $n>40$.

 

Nếu $n=41$, 

Không mất tính tổng quát, giả sử các số $a_{1}$, $a_{2}$,...,$a_{k}$ là các số âm và $a_{k+1}$, $a_{k+2}$,..., $a_{41}$ là các số dương và $k\leqslant 20$.

 

Do $\sum_{i=1}^{41} a_{i}=0$ nên $\left | \sum_{i=1}^{k} a_{i} \right |=\sum_{i=k+1}^{41} a_{i}<20$.

$\Rightarrow \sum_{i=1}^{k} a_{i}^{2}<20$ (1)

 

Mặt khác, do $a_{i} \in (-1;1)$ nên $a_{i}^{2} < \left |a_{i} \right|$.

$\Rightarrow \sum_{i=k+1}^{41} a_{i}^{2}< \sum_{i=k+1}^{41} a_{i}<20$ (2)

Từ $(1)$ và $(2)$, ta có được $\sum_{i=1}^{41} a_{i}^{2}<40$ (Mâu thuẫn). 

 

Do đó, với lí luận như trên, ta thấy $n \geqslant 42$. Do đó, giá trị nhỏ nhất của $n$ là $42$.


$\lim_{x \to \infty } Love =+\infty$


#39
Nguyenhuyen_AG

Nguyenhuyen_AG

    Trung úy

  • Thành viên nổi bật 2016
  • 945 Bài viết

Bài 18 (Macedonia National Olympiad). Cho $n \;(n \geqslant 3)$ số thực dương $a_1,\,a_2,\,\ldots,a_n$ thỏa mãn điều kiện

$$\frac{1}{1+a_1^4} + \frac{1}{1+a_2^4} + \cdots + \frac{1}{1+a_n^4} = 1.$$

Chứng minh rằng $$a_1a_2 \cdots a_n \ge (n-1)^{\frac n4}.$$
 


Nguyen Van Huyen
Ho Chi Minh City University Of Transport

#40
Ankh

Ankh

    Hạ sĩ

  • Thành viên
  • 85 Bài viết

Bài 18 (Macedonia National Olympiad). Cho $n \;(n \geqslant 3)$ số thực dương $a_1,\,a_2,\,\ldots,a_n$ thỏa mãn điều kiện

$$\frac{1}{1+a_1^4} + \frac{1}{1+a_2^4} + \cdots + \frac{1}{1+a_n^4} = 1.$$

Chứng minh rằng $$a_1a_2 \cdots a_n \ge (n-1)^{\frac n4}.$$
 

 Bài này quen thuộc

 Áp dụng bất đẳng thức AM-GM ta có $\dfrac{a_1^4}{1+a_1^4}=\dfrac{1}{1+a_2^4}+\dfrac{1}{1+a_3^4}+...+\dfrac{1}{a_n^4}\geq (n-1)\sqrt[n-1]{\dfrac{1}{(1+a_2^4)(1+a_3^4)...(1+a_n^4)}}$

 Tương tự thì $\begin{matrix}\dfrac{a_2^4}{1+a_2^4}\geq (n-1)\sqrt[n-1]{\dfrac{1}{(1+a_1^4)(1+a_3^4)...(1+a_n^4)}}\\ \dfrac{a_3^4}{1+a_3^4}\geq (n-1)\sqrt[n-1]{\dfrac{1}{(1+a_1^4)(1+a_2^4)...(1+a_n^4)}} \\... \\ \dfrac{a_n^4}{1+a_n^4}\geq (n-1)\sqrt[n-1]{\dfrac{1}{(1+a_1^4)(1+a_2^4)...(1+a_{n-1}^4)}}\end{matrix}$

 Nhân các bất đẳng thức trên lại ta được $\prod _{i=1}^n \dfrac{a_i^4}{1+a_i^4}\geq (n-1)^n.\dfrac{1}{\prod \limits _{i=1}^n (1+a_i^4)}$

 Thu gọn và lấy căn 2 vế ta có điều cần chứng minh






2 người đang xem chủ đề

0 thành viên, 2 khách, 0 thành viên ẩn danh